Montrer une inégalité — Les-mathematiques.net The most powerful custom community solution in the world

Montrer une inégalité

Bonjour
Soient $x_1,x_2...x_n$ des réels strictement positifs; j'essaie de montrer :
$$
\frac{x_1}{x_2} + \frac{x_2}{x_3} +\cdots+ \frac{x_{n-1}}{x_n} + \frac{x_n}{x_1} \geq n.

$$ J'ai pensé aux inégalités de convexité mais je suis bloqué... est-ce une bonne piste ?
Bonne journée.

Réponses

  • Inégalité arithmético-géométrique.
  • @side : c'est quoi stp ?

    @bisam : oui j'ai essayé d'appliquer avec $\log$, mais je suis embêté avec l'exposant $1/n$ :-S
  • @side : désolé je ne connais pas non plus :-S

    Par contre l'inégalité arithmético-géométrique ça je connais...
  • @Chalk : merci ! c'est pratique dis donc ce truc ! :-D
  • totem :
    \[\frac{1}{n}\left(\frac{x_1}{x_2}+\cdots+\frac{x_{n-1}}{x_n}+\frac{x_{n}}{x_1}\right)\geq \left(\frac{x_1}{x_2}\times\cdots\times\frac{x_{n-1}}{x_n}\times\frac{x_{n}}{x_1}\right)^{\frac{1}{n}}=1\]
  • @bisam : pfff...pourquoi ça paraît toujours évident une fois qu'on a la correction sous le nez ? :-X

    Merci, bien sûr :-)

    En tout cas, c'est carrément plus simple qu'avec l'inégalité de réarrangement il me semble !
  • Bof, l'inégalité de réarrangement est quand même plus compliquée à montrer que l'inégalité arithmético-géométrique !
  • Difficile de faire plus simple que l'inégalité de réarrangement, $\frac{x_1}{x_2} + \frac{x_2}{x_3} +\cdots+ \frac{x_{n-1}}{x_n} + \frac{x_n}{x_1} \geq \frac{x_1}{x_1} + \frac{x_2}{x_2} +\cdots+ \frac{x_{n}}{x_n} = n$, par contre oui la démonstration est un peu plus longue bien qu'élémentaire.
  • Pour l'inégalité de réarrangement, je veux bien croire que ce soit plus simple,... une fois qu'on la connait !

    Mais elle n'est pas (plus ?) au programme de prépa contrairement à l'autre...à moins que ?
  • À mon époque elle n'était effectivement pas au programme de prépa. Je n'ai découvert cette inégalité que récemment d'ailleurs, et c'est bien dommage !

    La démonstration paraît longue mais l'idée est fort simple, on vérifie que c'est vrai pour $n=2$, et ensuite on généralise, soit par récurrence, qui utilise le cas $n=2$ pour passer de $n$ à $n+1$, soit par une considération directe d'une permutation qui réalise le max, mais ce n'est pas plus naturel.
  • Edit : Ce qui suit comporte une erreur.

    Bonjour,
    Chalk l'inégalité de réarrangement ne donne pas $$\frac{x_1}{x_2} + \frac{x_2}{x_3} +\cdots+ \frac{x_{n-1}}{x_n} + \frac{x_n}{x_1} \geqslant \frac{x_1}{x_1} + \frac{x_2}{x_2} +\cdots+ \frac{x_{n}}{x_n}$$ mais $$\frac{x_1}{x_2} + \frac{x_2}{x_3} +\cdots+ \frac{x_{n-1}}{x_n} + \frac{x_n}{x_1} \geqslant \frac{x_{\sigma(1)}}{x_{\sigma(n)}} +\frac{x_{\sigma(2)}}{x_{\sigma(n-1)}}+\cdots+\frac{x_{\sigma(n-1)}}{x_{\sigma(2)}}+ \frac{x_{\sigma(n)}}{x_{\sigma(1)}}$$ où $\sigma\in{\mathfrak S}_n$ est telle que $x_{\sigma(1)}\leqslant x_{\sigma(2)}\leqslant \cdots\leqslant x_{\sigma(n)}$. Ensuite, il faut une étape supplémentaire pour conclure.
  • Oui tout à fait, étape triviale mais étape tout de même, donc au final je rejoins Poirot, l'inégalité artihmético-géométrique est plus directe dans ce cas !
  • Je me suis trompé dans mon message précédent. C'est $\frac{x_1}{x_2} + \frac{x_2}{x_3} +\cdots+ \frac{x_{n-1}}{x_n} + \frac{x_n}{x_1} \leqslant \frac{x_{\sigma(1)}}{x_{\sigma(n)}} +\frac{x_{\sigma(2)}}{x_{\sigma(n-1)}}+\cdots+\frac{x_{\sigma(n-1)}}{x_{\sigma(2)}}+ \frac{x_{\sigma(n)}}{x_{\sigma(1)}}$ et pas $\dots\geqslant\dots$. Donc cette inégalité là ne sert à rien ici et il n'y avait rien à corriger dans le message de Chalk.
    Pourtant c'est bien l'inégalité de réarrangement telle que je la connais et telle que la donne Wikipédia :
    Si $x_1 \leqslant x_2 \leqslant \dots \leqslant x_n$ et si $ y_1 \leqslant y_2 \leqslant \dots \leqslant y_n,$ alors $\displaystyle \forall \sigma\in\mathfrak{S}_n,\ \sum_{i=1}^nx_iy_i \geqslant \sum_{i=1}^nx_iy_{\sigma(i)}$.

    Mais side utilise
    Si $x_1 \leqslant x_2 \leqslant \dots \leqslant x_n$ et si $ y_1 \leqslant y_2 \leqslant \dots \leqslant y_n,$ alors $\displaystyle \forall \sigma\in\mathfrak{S}_n,\ \sum_{i=1}^nx_iy_{n+1-i} \leqslant \sum_{i=1}^nx_iy_{\sigma(i)}$.

    Peut-être qu'il y a des variantes de inégalité de réarrangement.
  • Encore des permutations ?! (:D
  • side a écrit:
    Autant dire que inégalité est triviale.

    Ton inégalité de réarrangement ? Euh... il ne faut pas exagérer.

    D'ailleurs, petite anecdote, j'ai donné en colle en MPSI l'inégalité de réarrangement il y a un mois et l'élève a proposé la preuve suivante : pour maximiser la somme $\sum_{i=1}^nx_iy_{\sigma(i)}$ avec $\sigma$ libre, on commence par former le plus grand produit, c'est $x_ny_n$, donc on pose $\sigma_\max(n)=n$, puis on forme le plus grand produit parmi ce qui reste i.e. $x_{n-1}y_{n-1}$, donc on pose $\sigma_\max(n-1)=n-1$, etc. Donc $\sum_{i=1}^nx_iy_i$ est maximal parmi les $\sum_{i=1}^nx_iy_{\sigma(i)}$.
    Sauf que... Appliquons le même raisonnement pour minimiser $\sum_{i=1}^nx_iy_{\sigma(i)}$. On commence par former le plus petit produit i.e. $x_1y_1$, donc $\sigma_\min(1)=1$, puis on forme le plus petit produit parmi ce qui reste i.e. $x_2y_2$, etc. Donc $\sum_{i=1}^nx_iy_i$ est minimal parmi les $\sum_{i=1}^nx_iy_{\sigma(i)}$.
    On a montré la propriété et son contraire ! :-D
  • totem a écrit:
    En tout cas, c'est carrément plus simple QU'avec l'inégalité de réarrangement il me semble !
    Poirot a écrit:
    Bof, l'inégalité de réarrangement est quand même plus compliquée à montrer que l'inégalité arithmético-géométrique !

    J'ai l'impression que vous ne vous êtes pas compris (NB: c'est moi qui mets en majuscules).
  • @Calli : oui c'est bien l'impression que j'avais, mais oserais-je défier un administrateur...:-P
  • Ah oui j'avais mal lu !
  • @Calli

    Sauf erreur, ce que tu as appelé « inégalité de side » est mentionné sur Wikipédia FR sous le théorème encadré, juste après « On a un résultat similaire pour le minimum de l'application ».
  • L'inégalité de réarrangement me gêne ici, car on a une hypothèse d'ensembles ordonnés sur les $x_i$ et les $y_î$, hypothèse que l'on n'a pas dans la question que j 'ai posée...du coup cela ne me parle pas...mais c'est sans doute mon humble niveau qui fait fait obstacle :-D
  • $]0, +\infty[$ n'est-il pas un ensemble ordonné pour la relation d'ordre qui intervient dans ta question ? 8-)
  • Effectivement brian. Je n'ai fait que parcourir rapidement la page Wikipédia en regardant les théorèmes encadrés. 8-)
  • @Poirot: effectivement, vu comme ça :-D
  • Une autre méthode.

    Soit a/x + x/b, x variable. On montre facilement que le min est obtenu avec x = V(ab). On note que V(ab) est compris entre a et b, en fait < (a+b)/2.

    On considère maintenant la suite des réels x1,x2,...xn et on cherche à rendre minimale la sommation. On remplace la plus grande valeur par le min ci-dessus, ce qui fait automatiquement abaisser le plus grand réel de la suite. En opérant ainsi une infinité de fois, on obtient une valeur unique pour tous les réels qui est égale au plus petit d'entre eux. Il est alors évident que la sommation minimale vaut n.
  • Bonsoir,
    @nogdim: on peut de cette façon le montrer par récurrence pour n pair du moins.
    On obtient le résultat pour n impair par déduction.
    Cordialement.
Connectez-vous ou Inscrivez-vous pour répondre.
Success message!